Which valve lesion typically produces a murmur of equal intensity throughout systole?
- A. Aortic stenosis
- B. Mitral insufficiency
- C. Pulmonic stenosis
- D. Aortic insufficiency
Correct Answer: C
Rationale: The correct answer is C: Pulmonic stenosis. This valve lesion typically produces a murmur of equal intensity throughout systole due to the obstruction of blood flow during both systole and diastole. Pulmonic stenosis results in turbulent blood flow across the pulmonary valve, creating a consistent murmur throughout the cardiac cycle.
Rationale for incorrect choices:
A: Aortic stenosis - Produces a crescendo-decrescendo murmur that peaks in mid-systole.
B: Mitral insufficiency - Causes a holosystolic murmur that is loudest at the apex.
D: Aortic insufficiency - Results in a diastolic murmur heard best at the left sternal border.
In summary, pulmonic stenosis is the correct answer as it uniquely presents with a murmur of equal intensity throughout systole, distinguishing it from the other valve lesions.
You may also like to solve these questions
In which of the following patients is chemical or surgical castration likely to prolong survival?
- A. A 78-year-old man with advanced dementia and prostate cancer metastatic to the bone.
- B. A 78-year-old man who had a radical prostatectomy 10 years earlier and now has a PSA level of 5.7 ng/mL. A year ago it was 0.1 ng/mL. A bone scan is negative and an abdomen-pelvis computed tomography (CT) is negative.
- C. A 78-year-old man who had a radical prostatectomy and external beam radiation therapy 10 years earlier now has a PSA level of 24.5 ng/mL. A CT scan of the pelvis shows an enlarged pelvic lymph node, and a bone scan is positive in the pelvis.
- D. A 78-year-old man with prostate cancer limited to the prostate with a Gleason score of 6.
Correct Answer: C
Rationale: The correct answer is C. In this scenario, the patient had a radical prostatectomy and external beam radiation therapy, indicating aggressive treatment. Despite this, the PSA level is high (24.5 ng/mL), suggesting biochemical recurrence and possible metastasis. The positive bone scan and enlarged pelvic lymph node indicate disease progression beyond the prostate. Chemical or surgical castration, such as androgen deprivation therapy, can help delay disease progression and prolong survival by reducing the levels of testosterone that fuel prostate cancer growth.
Summary:
A: In advanced dementia and metastatic bone disease, castration is unlikely to impact survival as the disease is already advanced.
B: A rising PSA level post-radical prostatectomy with negative imaging indicates biochemical recurrence, but without evidence of metastasis, making castration less beneficial.
D: Prostate cancer limited to the prostate with a Gleason score of 6 is low risk and may not benefit significantly from castration as it is less likely to progress rapidly.
A 12-year-old presents to the clinic with his father for evaluation of a painful lump in the left eye. It started this morning. He denies any trauma or injury. There is no visual disturbance. Upon physical examination, there is a red raised area at the margin of the eyelid that is tender to palpation; no tearing occurs with palpation of the lesion. Based on this description, what is the most likely diagnosis?
- A. Dacryocystitis
- B. Chalazion
- C. Hordeolum
- D. Xanthelasma
Correct Answer: C
Rationale: The correct answer is C: Hordeolum. A hordeolum, also known as a stye, presents as a red raised tender area at the margin of the eyelid. In this case, the absence of tearing upon palpation of the lesion suggests it is localized to the eyelid itself. Dacryocystitis (choice A) involves the lacrimal sac and would present with tearing. A chalazion (choice B) is a painless nodule on the eyelid, unlike the tender lesion described here. Xanthelasma (choice D) is characterized by yellowish plaques on the eyelids, not a red raised tender area.
All of the statements about Medicare are true except:
- A. All of the funding for Medicare comes from federal taxes.
- B. Durable medical equipment has no out-of-pocket cost to the patient.
- C. Part B covers outpatient services and all physician professional fees whether inpatient or outpatient.
- D. Patients are generally responsible for 20% of Part B bills.
Correct Answer: B
Rationale: The correct answer is B because durable medical equipment under Medicare typically requires the patient to pay 20% of the Medicare-approved amount.
A is incorrect because Medicare funding also comes from payroll taxes and individual premiums.
C is incorrect as Part B covers outpatient services and physician fees only in outpatient settings.
D is incorrect as patients are generally responsible for 20% of Part B bills, not no out-of-pocket cost for durable medical equipment.
A 33-year-old construction worker comes for evaluation and treatment of acute onset of low back pain. He notes that the pain is an aching located in the lumbosacral area. It has been present intermittently for several years; there is no known trauma or injury. He points to the left lower back. The pain does not radiate and there is no numbness or tingling in the legs or incontinence. He was moving furniture for a friend over the weekend. On physical examination, you note muscle spasm, with normal deep tendon reflexes and muscle strength. What is the most likely cause of this patient's low back pain?
- A. Herniated disc
- B. Compression fracture
- C. Mechanical low back pain
- D. Ankylosing spondylitis
Correct Answer: C
Rationale: The correct answer is C: Mechanical low back pain. This is the most likely cause as the patient's pain is aching, located in the lumbosacral region, intermittent for years, with no radiation, numbness, tingling, or incontinence. The physical exam findings of muscle spasm, normal reflexes, and muscle strength support this diagnosis. Mechanical low back pain is common and often related to muscle strain or overuse, as seen in this construction worker who was moving furniture.
Choice A (Herniated disc) is incorrect as the patient does not have radiation of pain or neurological symptoms. Choice B (Compression fracture) is unlikely without a history of trauma or injury. Choice D (Ankylosing spondylitis) is less likely given the lack of inflammatory symptoms and typical age of onset.
Why is it important to inquire about the age of the first period when taking a sexual history from female patients?
- A. It helps to determine the patient's current level of physical fitness.
- B. It provides insight into the patient's reproductive health and potential risks.
- C. It indicates the patient's dietary habits.
- D. It is irrelevant to sexual health assessments.
Correct Answer: B
Rationale: The correct answer is B because knowing the age of the first period can give insight into the patient's reproductive health and potential risks. Early or late onset of menstruation can be indicators of hormonal imbalances, reproductive disorders, or potential fertility issues. This information is crucial for assessing the patient's overall gynecological health and guiding appropriate interventions.
Choice A is incorrect because the age of the first period is not directly related to determining the patient's current level of physical fitness. Choice C is incorrect because the age of the first period does not indicate the patient's dietary habits. Choice D is incorrect because the age of the first period is relevant to sexual health assessments as it provides valuable information about the patient's reproductive health.